The series $\sum a_n$ is the series I get when analyzing the endpoint of a power series, so the ratio test is out of the question. How else can I find whether this particular series converges?
-
1Keyword: Stirling's approximation (+limit comparison test). – Clement C. Nov 29 '19 at 02:13
-
WIthout using it? This is supposed to be a very basic calculus class so I'm forbidden from using it – Francisco José Letterio Nov 29 '19 at 02:14
-
So then, again, why is the ratio test out of the question? – Clement C. Nov 29 '19 at 02:15
-
because it's the endpoint of the radius of convergence of a power series. The ratio test yields 1 – Francisco José Letterio Nov 29 '19 at 02:16
-
Admittedly, but you have the usual extensions. – Clement C. Nov 29 '19 at 02:17
-
and how do I use that extension? – Francisco José Letterio Nov 29 '19 at 02:19
-
Voting to reopen: The answers to the question linked (for those having marked it as duplicate) use Stirling's or more advanced ideas, which the OP said was not allowed. (Maybe be less quick to close?) – Clement C. Nov 29 '19 at 02:32
-
For what it's worth, the OP seems to have accepted the duplicate. Also, this is a question that has been asked a lot of times already (see the list of questions linked to the duplicate target), and it's getting hard to compare all the answers. I think it would be better to keep this closed, but I admit that I may not have chosen the best target (and now the target has been closed as a duplicate itself). – Arnaud D. Nov 29 '19 at 15:10
-
@ClementC.For what it's worth , the duplicate of the current duplicate, https://math.stackexchange.com/questions/177538/finding-the-convergence-interval-of-sum-limits-n-0-infty-fracnxnnn has the simple solution via the series embedded in an Edit of the question itself – Hagen von Eitzen Nov 30 '19 at 08:29
1 Answers
What's below is overkill, as pointed out in the comments. Indeed, once it's established that $$ \frac{a_{n+1}}{a_n} = \frac{e}{(1+\frac{1}{n})^{n}} > 1 $$ then it's clear that the sequence $(a_n)_n$ is inctreasing. Therefore, $a_n \not\to 0$, and the series $\sum_n a_n$ trivially diverges by the limit test.
Since Stirling's approximation is not allowed*, let's try extensions of the ratio test.
Since $$ \frac{a_{n+1}}{a_n} = \frac{(n+1)e\cdot n^n}{(n+1)^{n+1}} = \frac{e\cdot n^n}{(n+1)^{n}} = \frac{e}{(1+\frac{1}{n})^{n}} $$ we have $$\begin{align*} n\left( \frac{a_n}{a_{n+1}} -1 \right) &= e^{-1}n\left( \left(1+\frac{1}{n}\right)^{n} - e\right) = e^{-1}n\left( e^{n \ln\left(1+\frac{1}{n}\right)} - e\right)\\ &= e^{-1}n\left( e^{n(\frac{1}{n} - \frac{1}{2n^2}+o\left(\frac{1}{n^2}\right))} - e\right)\\ &= e^{-1}n\left( e^{1-\frac{1}{2n}+O\left(\frac{1}{n^2}\right)} - e\right) = n\left( e^{-\frac{1}{2n}+O\left(\frac{1}{n^2}\right)} - 1\right)\\ &= n\left( -\frac{1}{2n}+O\left(\frac{1}{n^2}\right)\right)\\ &\xrightarrow[n\to\infty]{} \boxed{-\frac12} < 1 \end{align*}$$ so the series diverges by Raabe's test.
${}^\ast$ Of course, if you want to use it: $n!\displaystyle\operatorname*{\sim}_{n\to\infty} {\sqrt{2\pi n}}\frac{n^n}{e^n}$, so $$ a_n \operatorname*{\sim}_{n\to\infty} {\sqrt{2\pi n}} $$ and the series $\sum_n a_n$ (of positive terms) diverges by the limit test.

- 67,323
-
sorry but raabe's test and the other approximation are also out of the question – Francisco José Letterio Nov 29 '19 at 02:34
-
@FranciscoJoséLetterio That's fair.It'd be worth editing your question to make that clear (esp. as otherwise, it's likely to get closed as it's smilar to other questions without those constraints). – Clement C. Nov 29 '19 at 02:47
-
1In Stirling, $\sqrt{2\pi n}$ is in the numerator, not the denominator. Here, $a_{n+1}/a_n>1$, so it's apparent the series diverges. A more challenging series (again restricting to "elementary" methods) is $\sum_n\frac{n^n}{e^nn!}$. – Angina Seng Nov 29 '19 at 03:18
-
@LordSharktheUnknown My bad yes, the Stirling approach is even more obvious than expected. And yes, knowing that $(1+1/n)^n < e$, Raabe's test and the ratio test are both unnecessary -- the ratio is greater that one, so the sequence $(a_n)_n$ is increasing, and therefore the series diverges by the limit test. – Clement C. Nov 29 '19 at 03:24
-